Re: [Bulk] [obm-l] Re: [obm-l] Congruências com primos

2016-08-18 Por tôpico Listeiro 037

Será que existe um texto sobre isto?

Em Thu, 18 Aug 2016 11:31:54 -0300
Anderson Torres  escreveu:

> A ideia é que 1/N mod p seja a solução da "equação" Nx=1 (mod p).
> 
> Em 3 de agosto de 2016 18:15, Israel Meireles Chrisostomo
>  escreveu:
> > Olá pessoal já estudei um pouco de congruências, mas não sei muito
> > bem em como lidar com congruências fracionárias.Por exemplo, alguém
> > poderia me explicar o pq da congruência abaixo?
> >
> > Seja p um primo
> >
> > então podemos dizer que 1/(p-1)≡1/-1≡-1(mod p)
> >
> > 1/(p-2)≡1/-2≡-1/2(mod p)
> > 1/(p-3)≡1/-3≡-1/3(mod p)
> > 
> > 1/(p-k)≡1/-k≡-1/k(mod p)
> >
> > como explicar isso?
> >
> > --
> > Esta mensagem foi verificada pelo sistema de antivírus e
> > acredita-se estar livre de perigo.
> 

-- 
Esta mensagem foi verificada pelo sistema de antiv�rus e
 acredita-se estar livre de perigo.


=
Instru��es para entrar na lista, sair da lista e usar a lista em
http://www.mat.puc-rio.br/~obmlistas/obm-l.html
=


[obm-l] Re: [obm-l] Re: [obm-l] Congruências com primos

2016-08-18 Por tôpico Israel Meireles Chrisostomo
Muito obrigado

Em 18 de agosto de 2016 11:31, Anderson Torres  escreveu:

> A ideia é que 1/N mod p seja a solução da "equação" Nx=1 (mod p).
>
> Em 3 de agosto de 2016 18:15, Israel Meireles Chrisostomo
>  escreveu:
> > Olá pessoal já estudei um pouco de congruências, mas não sei muito bem em
> > como lidar com congruências fracionárias.Por exemplo, alguém poderia me
> > explicar o pq da congruência abaixo?
> >
> > Seja p um primo
> >
> > então podemos dizer que 1/(p-1)≡1/-1≡-1(mod p)
> >
> > 1/(p-2)≡1/-2≡-1/2(mod p)
> > 1/(p-3)≡1/-3≡-1/3(mod p)
> > 
> > 1/(p-k)≡1/-k≡-1/k(mod p)
> >
> > como explicar isso?
> >
> > --
> > Esta mensagem foi verificada pelo sistema de antivírus e
> > acredita-se estar livre de perigo.
>
> --
> Esta mensagem foi verificada pelo sistema de antivírus e
>  acredita-se estar livre de perigo.
>
>
> =
> Instru�ões para entrar na lista, sair da lista e usar a lista em
> http://www.mat.puc-rio.br/~obmlistas/obm-l.html
> =
>

-- 
Esta mensagem foi verificada pelo sistema de antiv�rus e
 acredita-se estar livre de perigo.



[obm-l] Re: [obm-l] Congruências com primos

2016-08-18 Por tôpico Anderson Torres
A ideia é que 1/N mod p seja a solução da "equação" Nx=1 (mod p).

Em 3 de agosto de 2016 18:15, Israel Meireles Chrisostomo
 escreveu:
> Olá pessoal já estudei um pouco de congruências, mas não sei muito bem em
> como lidar com congruências fracionárias.Por exemplo, alguém poderia me
> explicar o pq da congruência abaixo?
>
> Seja p um primo
>
> então podemos dizer que 1/(p-1)≡1/-1≡-1(mod p)
>
> 1/(p-2)≡1/-2≡-1/2(mod p)
> 1/(p-3)≡1/-3≡-1/3(mod p)
> 
> 1/(p-k)≡1/-k≡-1/k(mod p)
>
> como explicar isso?
>
> --
> Esta mensagem foi verificada pelo sistema de antivírus e
> acredita-se estar livre de perigo.

-- 
Esta mensagem foi verificada pelo sistema de antiv�rus e
 acredita-se estar livre de perigo.


=
Instru��es para entrar na lista, sair da lista e usar a lista em
http://www.mat.puc-rio.br/~obmlistas/obm-l.html
=


[obm-l] Re: [obm-l] Congruências

2015-09-25 Por tôpico Pedro José
Bom dia!

Dados dois inteiros *d* e *a* dizemos que:

d divide a, ou d é divisor de a ou a é múltiplo de d e representamos por  d
| a <==> Existe k Ɛ Z | kd = a.
Portanto, pela definição, se b | |a| ==. Existe k inteiro tal que  kb = |a|.

Se a >= 0 ==> |a| = a ==> kb = a ==> b | a.
Se a <0 ==> |a| = - a ==> Existe  w=-k, w inteiro, tal que wb= a ==> b | a.

Atentar que não existe a/0, porém pela definição 0 | 0.

Saudações,
PJMS


Em 25 de setembro de 2015 03:01, Israel Meireles Chrisostomo <
israelmchrisost...@gmail.com> escreveu:

> ola pessoal, se b divide |a| então b divide a?isso me parece meio óbvio,
> de fato parece ser verdadeiro,mas mesmo assim gostaria de uma resposta...
>
> --
> Esta mensagem foi verificada pelo sistema de antivírus e
> acredita-se estar livre de perigo.

-- 
Esta mensagem foi verificada pelo sistema de antiv�rus e
 acredita-se estar livre de perigo.



[obm-l] Re: [obm-l] Congruências

2015-09-25 Por tôpico Cassio Anderson Feitosa
Sim. b divide |a| ==> |a|=kb ==> a=kb ou a=(-k)b.

Em 25 de setembro de 2015 03:01, Israel Meireles Chrisostomo <
israelmchrisost...@gmail.com> escreveu:

> ola pessoal, se b divide |a| então b divide a?isso me parece meio óbvio,
> de fato parece ser verdadeiro,mas mesmo assim gostaria de uma resposta...
>
> --
> Esta mensagem foi verificada pelo sistema de antivírus e
> acredita-se estar livre de perigo.




-- 
Cássio Anderson
Graduando em Matemática - UFPB

-- 
Esta mensagem foi verificada pelo sistema de antiv�rus e
 acredita-se estar livre de perigo.



[obm-l] Re: [obm-l] Congruências

2015-09-25 Por tôpico Cassio Anderson Feitosa
Na verdade, |a|=kb ===> |a|=|kb| ===>   a=kb ou a=(-k)b.

Em 25 de setembro de 2015 10:33, Cassio Anderson Feitosa <
cassiofeito...@gmail.com> escreveu:

> Sim. b divide |a| ==> |a|=kb ==> a=kb ou a=(-k)b.
>
> Em 25 de setembro de 2015 03:01, Israel Meireles Chrisostomo <
> israelmchrisost...@gmail.com> escreveu:
>
>> ola pessoal, se b divide |a| então b divide a?isso me parece meio óbvio,
>> de fato parece ser verdadeiro,mas mesmo assim gostaria de uma resposta...
>>
>> --
>> Esta mensagem foi verificada pelo sistema de antivírus e
>> acredita-se estar livre de perigo.
>
>
>
>
> --
> Cássio Anderson
> Graduando em Matemática - UFPB
>



-- 
Cássio Anderson
Graduando em Matemática - UFPB

-- 
Esta mensagem foi verificada pelo sistema de antiv�rus e
 acredita-se estar livre de perigo.



[obm-l] RE: [obm-l] Re: [obm-l] Congruências

2014-02-05 Por tôpico marcone augusto araújo borges
Já apliquei o teorema e funcionou.Obrigado!

Date: Tue, 4 Feb 2014 17:48:38 -0200
Subject: [obm-l] Re: [obm-l] Congruências
From: ralp...@gmail.com
To: obm-l@mat.puc-rio.br

Sendo x esse numero, voce descobriu que x-1 eh multiplo de 7, de 11 e de 13.
Como eles sao primos, entao x-1 eh multiplo de 7.11.13 = 1001. Entao voce tem 
razao: x deixa resto 1 na divisao por 1001.

Uma generalizacao desta ideia eh o Teorema Chines do Resto:
http://pt.wikipedia.org/wiki/Teorema_chin%C3%AAs_do_resto

Ou Teorema do Resto Chines? :) :) 
:)http://en.wikipedia.org/wiki/Chinese_remainder_theorem


Abraco,Ralph

2014-02-04 marcone augusto araújo borges marconeborge...@hotmail.com:




Determinar o resto da divisão de 300^3000 por 1001
 
 
Pelos meus cálculos essa potência dividida por 7,por 11
ou por 13 deixa o mesmo resto 1
como 7,11 e 13 são primos e 7.11.13 = 1001,posso afirmar

300^3000 dividido por 1001 deixa resto 1?
  
--

Esta mensagem foi verificada pelo sistema de antivírus e 

 acredita-se estar livre de perigo.





--

Esta mensagem foi verificada pelo sistema de antivírus e 

 acredita-se estar livre de perigo.   
-- 
Esta mensagem foi verificada pelo sistema de antivírus e
 acredita-se estar livre de perigo.



[obm-l] Re: [obm-l] Congruências

2014-02-04 Por tôpico saulo nilson
300^1=300MOD1001
300^2=911MOD1001
300^3=27MOD1001
 =92MOD1001
=573MOD1001
==729MOD1001
482MOD1001
456MOD1001
664
1MOD1001
COMO 3000 E MULTIPLO DE 10
ENTAO
300^3000=1MOD1001


2014-02-04 marcone augusto araújo borges marconeborge...@hotmail.com:

 Determinar o resto da divisão de 300^3000 por 1001


 Pelos meus cálculos essa potência dividida por 7,por 11
 ou por 13 deixa o mesmo resto 1
 como 7,11 e 13 são primos e 7.11.13 = 1001,posso afirmar
 300^3000 dividido por 1001 deixa resto 1?

 --
 Esta mensagem foi verificada pelo sistema de antivírus e
 acredita-se estar livre de perigo.


-- 
Esta mensagem foi verificada pelo sistema de antivírus e
 acredita-se estar livre de perigo.



[obm-l] Re: [obm-l] Congruências

2014-02-04 Por tôpico Ralph Teixeira
Sendo x esse numero, voce descobriu que x-1 eh multiplo de 7, de 11 e de 13.

Como eles sao primos, entao x-1 eh multiplo de 7.11.13 = 1001. Entao voce
tem razao: x deixa resto 1 na divisao por 1001.

Uma generalizacao desta ideia eh o Teorema Chines do Resto:

http://pt.wikipedia.org/wiki/Teorema_chin%C3%AAs_do_resto

Ou Teorema do Resto Chines? :) :) :)
http://en.wikipedia.org/wiki/Chinese_remainder_theorem

Abraco,
Ralph


2014-02-04 marcone augusto araújo borges marconeborge...@hotmail.com:

 Determinar o resto da divisão de 300^3000 por 1001


 Pelos meus cálculos essa potência dividida por 7,por 11
 ou por 13 deixa o mesmo resto 1
 como 7,11 e 13 são primos e 7.11.13 = 1001,posso afirmar
 300^3000 dividido por 1001 deixa resto 1?

 --
 Esta mensagem foi verificada pelo sistema de antivírus e
 acredita-se estar livre de perigo.


-- 
Esta mensagem foi verificada pelo sistema de antivírus e
 acredita-se estar livre de perigo.



Re: [obm-l] Re: [obm-l] congruências

2003-12-30 Por tôpico Luiz Ponce




 isso mesmo
Muito obrigado
Claudio Freitas,
PONCE

Claudio Freitas escreveu:
  
  
 
  
 

  Acho que  porque..
 
  n^5 - n = n (n^ 4 - 1) = n ( n ^2 -
1  ) (n^2 + 1)  [  1 ]
 
  n ( n ^ 2 - 1 ) ( n ^  2 + 1) = n ( n ^ 2 - 1)[( n ^ 2 - 4) +  5]
 
  = n ( n ^ 2 - 1) (n ^ 2 -  4)
+ n ( n ^ 2 - 1)  (5 )
 
  
 
   
  
-
Original Message - 
   
From:
   [EMAIL PROTECTED]
   
   
To:
[EMAIL PROTECTED]

   
Sent:
Tuesday, December 30, 2003 1:16AM
   
Subject:
Re: [obm-l] congruncias
   


Para oproprio
Ponce ou alguem que saiba. Nao entendi uma passagem. 
Por que o 5estah sendo multiplicado por n( n ^2 - 1 ) ? Pois o 5 da segunda
equacao [2]nao estah ? 


Em uma mensagem de 30/12/2003 00:20:00 Hor. de veroleste da Am. Su,
[EMAIL PROTECTED]escreveu:



   
Caro
amigo Jefferson, 
Vai uma humilde sugesto . 
Da  definio de "
congruncia mod m"
, tem-se que: 
n^5   congruente a n ( mod 15) se, e somente se, n^5 - n  
divisivel por 15. 
  
Por outro lado, para todo n natural 
  
n^5  - n = n (n^ 4 - 1) = n ( n ^2 - 1 ) (n^2 + 1)  [
 1 ]   
  
n ^2 + 1 = (n ^2 - 4) + 5 = (n - 2 )(n + 2) +  5
 [  2 ] 
  
De [ 1 ] e [ 2 ] resulta 
n^5 - n = n ( n ^2 - 1  )(n - 2 )(n + 2) + 5.
n ( n ^2 - 1  )
 
ou melhor ainda 
n^5 - n = (n - 2 )(n-1)n(n+1)(n + 2) +  5. (n-1)n(n+1) 
Assim, n^5 - n = A + 5.B, onde  
A = (n - 2 )(n-1)n(n+1)(n + 2) (  produto de cinco inteiros consecutivos)
  
B = (n-1)n(n+1)  (  produto de tres inteiros
consecutivos) 
  
Lembrando que o produto de n  (n1) inteiros consecutivos  sempre
divisivel 
por n ! ( n  fatorial), tem- se que : 
A  divisivel por: 5 !, ou  seja 120 enquanto 5.B  divisivel por
5. 3! , ou  seja, 30 
  
Agora, como o MDC ( 120, 30) = 30, conclui-se  que A + 5B  divisivel
por 30 . 
  
Portanto, sendo  30 = 15. 2 , podemos afirmar que n^5 - n  divisivel
por  15, 
isto , n^5  congruente a n ( mod 15), o  que finaliza a demonstrao.
  
  
PONCE 
Nota: Da demonstrao acima,  resulta que :n^5  congruente a n (
mod 30).  
  
  
  
Jefferson Franca escreveu: 
 
  Ser q algum poderia dar uma mo com a questo:Prove q para
   um natural n , tem-se que n^5 congruente n ( mod 15)
  
  
  


   
   
 
 Esta mensagem foi verificada pelo
E-mail Protegido  
 Terra.
Scan engine: VirusScan / Atualizado em 24/12/2003 / Verso:1.4.1
Proteja o seu e-mail Terra: http://www.emailprotegido.terra.com.br/
  






Re: [obm-l] RE: [obm-l] congruências

2003-12-30 Por tôpico Jefferson Franca
Muito obrigado pela solução. 
Hoje de manhã fiquei pensando um pouco mais sobre esta questão e cheguei à seguinte idéia: se n^5 congruente n ( mod 15), então, n^5 - n deve ser múltiplo de 15, ou seja, deve ser múltiplo de 3 e de 5 ao mesmo tempo, observe que fatorando provamos isso: n^5 - n = (n-2)(n-1)n(n+1)(n+2) + 5(n-1)n(n+1). O primeiro termo tem 5 números consecutivos, então é múltiplo de 5 e de 3, o segundo termo 3 números consecutivos e além disso é múltiplo de 5, logo provamos que n^5 - n é múltiplo de 15 , ou seja, n^5 congruente n (mod 15)
Douglas Bokliang [EMAIL PROTECTED] wrote:
provar q n^5=n (mod 15) eh a mesma coisa q provar q n^5=n (mod 5) e n^5=n (mod 3)pelo peq. teor. Fermat:n^(p-1)=1 (mod p), com p primo e n nao multiplo de p1)n^4=1 (mod 5)n^5=n (mod 5)2)n^2=1 (mod 3)n^4=1 (mod 3)n^5=n (mod 3)para n multiplo de p, eh obvio q n^5=n (mod p)[]´sDouglas BokliangFrom: Jefferson Franca <[EMAIL PROTECTED]>Reply-To: [EMAIL PROTECTED]To: [EMAIL PROTECTED]Subject: [obm-l] congruênciasDate: Mon, 29 Dec 2003 18:36:43 -0300 (ART)Será q alguém poderia dar uma mão com a questão:Prove q para um natural n , tem-se que n^5 congruente n ( mod 15)-Central anti-spam do Yahoo! Mail: com dicas, dúvidas e
 curiosidades!_MSN Messenger: converse com os seus amigos online. http://messenger.msn.com.br=Instruções para entrar na lista, sair da lista e usar a lista emhttp://www.mat.puc-rio.br/~nicolau/olimp/obm-l.html=Central anti-spam do Yahoo! Mail: com dicas, dúvidas e curiosidades!

[obm-l] RE: [obm-l] congruências

2003-12-29 Por tôpico Douglas Bokliang
provar q n^5=n (mod 15) eh a mesma coisa q provar q n^5=n (mod 5) e n^5=n 
(mod 3)

pelo peq. teor. Fermat:
n^(p-1)=1 (mod p), com p primo e n nao multiplo de p
1)n^4=1 (mod 5)
n^5=n (mod 5)
2)n^2=1 (mod 3)
n^4=1 (mod 3)
n^5=n (mod 3)
para n multiplo de p, eh obvio q n^5=n (mod p)

[]´s
Douglas Bokliang
From: Jefferson Franca [EMAIL PROTECTED]
Reply-To: [EMAIL PROTECTED]
To: [EMAIL PROTECTED]
Subject: [obm-l] congruências
Date: Mon, 29 Dec 2003 18:36:43 -0300 (ART)
Será q alguém poderia dar uma mão com a questão:Prove q para um natural n , 
tem-se que n^5 congruente n ( mod 15)



-
Central anti-spam do Yahoo! Mail: com dicas, dúvidas e curiosidades!
_
MSN Messenger: converse com os seus amigos online.  
http://messenger.msn.com.br

=
Instruções para entrar na lista, sair da lista e usar a lista em
http://www.mat.puc-rio.br/~nicolau/olimp/obm-l.html
=


[obm-l] Re: [obm-l] congruências

2003-12-29 Por tôpico Webmaster - Cnaval



n^5 - n = n(n^4-1) = n(n^2 + 1)(n + 1)(n - 1) - 
 3 numeros consecutivos(n-1, n , n+1)- multiplo de 
3
basta agora vc provar que é multiplo de 5, usando o 
pequeno teorema de fermat fica imediato.
Outro jeito de vc provar que é multiplo de 5 eh vc 
ir substituindo...
se n = 5k (k inteiro) -  imediato
n = 5k + 1 - o termo n - 1 nos dá um multiplo 
de 5
n = 5k + 2 -  o termo n^2 + 1 nos dá um 
multiplo de 5
n = 5k + 3 -  o termo n^2 +1 nos dá um multiplo 
de 5
n = 5k + 4 -  o termo n + 1 nos dá um multiplo 
de 5
Como n só pode dar esses possíveis restos por 5. 
n^5 - n será multiplo de 5 e de 3, logo, multiplo de 15.
Acho que é isso, espero n ter escrito besteira. 
:P

Igor Castro
( www.cnaval.hog.com.br )


  --1-- Original Message - 
  From: 
  Jefferson 
  Franca 
  To: [EMAIL PROTECTED] 
  Sent: Monday, December 29, 2003 7:36 
  PM
  Subject: [obm-l] congruências
  
  Será q alguém poderia dar uma mão com a questão:Prove q para um natural n 
  , tem-se que n^5 congruente n ( mod 15)
  
  
  Central 
  anti-spam do Yahoo! Mail: com dicas, 
  dúvidas 
  e curiosidades!


[obm-l] Re: [obm-l] congruências

2003-12-29 Por tôpico Claudio Freitas



Acho que é porque..
n^5 - n = n (n^ 4 - 1) = n ( n ^2 - 1 
) (n^2 + 1) 
[ 
1 ]
n ( n ^ 2 - 1 ) ( n ^ 
2 + 1) = n ( n ^ 2 - 1)[( n ^ 2 - 4) + 
5]
= n ( n ^ 2 - 1) (n ^ 2 - 
4) + n ( n ^ 2 - 1) 
(5 )


  - Original Message - 
  From: 
  [EMAIL PROTECTED] 
  
  To: [EMAIL PROTECTED] 
  Sent: Tuesday, December 30, 2003 1:16 
  AM
  Subject: Re: [obm-l] congruências
  Para o 
  proprio Ponce ou alguem que saiba. Nao entendi uma passagem. Por que o 5 
  estah sendo multiplicado por n( n ^2 - 1 ) ? Pois o 5 da segunda equacao [2] 
  nao estah ? Em uma mensagem de 30/12/2003 00:20:00 Hor. de verão 
  leste da Am. Su, [EMAIL PROTECTED] 
  escreveu: 
  Caro amigo Jefferson, Vai uma humilde sugestão . Da 
definição de " congruência mod m" , tem-se que: n^5 
é congruente a n ( mod 15) se, e somente se, n^5 - n 
é divisivel por 15. Por outro lado, para todo n natural n^5 
- n = n (n^ 4 - 1) = n ( n ^2 - 1 ) (n^2 + 1) 
[ 
1 ]  
 
n ^2 + 1 = (n ^2 - 4) + 5 = (n - 2 )(n + 2) + 
5 
[ 
2 ] De [ 1 ] e [ 2 ] resulta n^5 - n = n ( n ^2 - 1 
)(n - 2 )(n + 2) + 5. n ( n ^2 - 1 
) 
ou melhor ainda n^5 - n = (n - 2 )(n-1)n(n+1)(n + 2) + 
5. (n-1)n(n+1) Assim, n^5 - n = A + 5.B, onde 
A = (n - 2 )(n-1)n(n+1)(n + 2) ( 
produto de cinco inteiros consecutivos) 
 B = (n-1)n(n+1) 
( 
produto de tres inteiros consecutivos) Lembrando que o produto de n 
(n1) inteiros consecutivos é sempre divisivel por n ! ( n 
fatorial), tem- se que : A é divisivel por: 5 !, ou 
seja 120 enquanto 5.B é divisivel por 5. 3! , ou 
seja, 30 Agora, como o MDC ( 120, 30) = 30, conclui-se 
que A + 5B é divisivel por 30 . Portanto, sendo 
30 = 15. 2 , podemos afirmar que n^5 - n é divisivel por 
15, isto é, n^5 é congruente a n ( mod 15), o 
que finaliza a demonstração. PONCE Nota: Da demonstração acima, 
resulta que :n^5 é congruente a n ( mod 30). 
Jefferson Franca escreveu: 
Será q alguém poderia dar uma mão com a questão:Prove q para 
  um natural n , tem-se que n^5 congruente n ( mod 15) 
  
  
  
  Esta mensagem foi verificada pelo E-mail Protegido 
  Terra.Scan engine: VirusScan / Atualizado em 24/12/2003 / Versão: 
  1.4.1Proteja o seu e-mail Terra: http://www.emailprotegido.terra.com.br/